Уравнение Дирака в общей теории относительности

Уравнение Дирака для безмассовых фермионов в искривленном пространственном времени имеет вид γ а е а мю Д мю Ψ "=" 0 , где е а мю это тетрады. Я должен показать, что спиноры Дирака подчиняются следующему уравнению:

( Д мю Д мю + 1 4 р ) Ψ "=" 0 ( 1 )

где р является скаляром Риччи.

я уже знаю, что [ Д мю , Д ν ] А р "=" р мю ν р о А о , но главное знать, что [ Д мю , Д ν ] Ψ является.

( Д мю Ψ "=" мю Ψ + А мю а б Σ а б является ковариантной производной спинорного поля и Σ а б генераторы Лоренца с гамма-матрицами).

Правильный способ решить эту проблему — действовать с Д мю слева в вашем уравнении Дирака GR снова. Как только вы это сделаете, вы должны понять, что новая ковариантная производная также действует на тетрады, что означает, что она эффективно дифференцирует метрику и в конечном итоге дает скалярный член Риччи. Однако вы должны понимать, что прилагательное «ковариантный» означает, что оно имеет два новых термина связи, один из метрики искривленного пространства-времени и один из электромагнитного потенциала. Последнее вообще не нужно для вашего желаемого вывода - калибровочное поле просто сохраняется везде как часть Д мю .
Связано это с physics.stackexchange.com/questions/51269/… Они действительно разные? Если нет, можно подумать о том, чтобы попросить модов объединить их.
Спасибо за комментарий, но Д мю е а ν не ноль?
Разве тетрады не являются ковариантно постоянными при условии, что вы включаете как символы Кристоффеля, так и спиновую связь?
Да, они! Можете ли вы написать явно, что вы имеете в виду для Д мю е а ν и как это может решить проблему... Теперь я немного запутался в правильном действии Д мю на тетрады.
Я бы сказал, что cov производная от тетрады что-то вроде мю е ν а Г мю ν о е о а + ю мю а б е ν б были ю мю а б является спиновой связью. (Греческое — пространство-время, а латынь — тетрадная метка, также я держу здесь все, что связано с электромагнитными датчиками). (PS я не утверждаю, что решил вашу проблему!)

Ответы (1)

Обозначая γ а гамма-матрицы пространства Минковского относительно тетрады Лоренца { е а } и ковариантная производная Д а , то гамма ковариантно постоянна.

Начните с безмассового уравнения Дирака

γ б Д б Ψ "=" 0

Действуйте снова с оператором Дирака

γ а Д а γ б Д б Ψ "=" 0
Итак, поскольку Д уничтожает γ
γ а γ б Д а Д б Ψ "=" 0
так
1 2 { γ а , γ б } Д а Д б Ψ + 1 2 γ а γ б [ Д а , Д б ] Ψ "=" 0     ( 1 )
Но
{ γ а , γ б } "=" 2 η а б
и
[ Д а , Д б ] Ψ "=" р а б Ψ
Где р а б — спиновая кривизна (антисимметричная по a и b). р а б удовлетворяет тождеству
γ б р а б "=" р а б γ б "=" 1 2 γ б р а б
где р а б — тензор Риччи (в тетраде Лоренца). поэтому (1) становится
[ Д а Д а + 1 4 γ а γ б р а б ] Ψ "=" 0
то есть
[ Д а Д а 1 4 р ] Ψ "=" 0

Ваш ответ вдохновил на дополнительный вопрос .